How to prove that the given lines are perpendicular?












1












$begingroup$


I have a triangle $triangle ABC$, with circumcentre $O$, the reflection of $A$ through $BC$ gives me $T$. The perpendicular bisector of $TC$ and $TB$ intersect $AB$ and $AC$ at $P$ and $Q$ respectively.



I need to prove that $TO$ is perpendicular to $PQ$.



How do I prove it? I tried some simple angle chasing but couldn't come to any significant conclusion. The solution booklet however tells me to use the fact that the reflection of $O$ in $BC$, say $O'$ lies on the circumcircle of $triangle APQ$, how did he arrive at this conclusion?










share|cite|improve this question











$endgroup$

















    1












    $begingroup$


    I have a triangle $triangle ABC$, with circumcentre $O$, the reflection of $A$ through $BC$ gives me $T$. The perpendicular bisector of $TC$ and $TB$ intersect $AB$ and $AC$ at $P$ and $Q$ respectively.



    I need to prove that $TO$ is perpendicular to $PQ$.



    How do I prove it? I tried some simple angle chasing but couldn't come to any significant conclusion. The solution booklet however tells me to use the fact that the reflection of $O$ in $BC$, say $O'$ lies on the circumcircle of $triangle APQ$, how did he arrive at this conclusion?










    share|cite|improve this question











    $endgroup$















      1












      1








      1





      $begingroup$


      I have a triangle $triangle ABC$, with circumcentre $O$, the reflection of $A$ through $BC$ gives me $T$. The perpendicular bisector of $TC$ and $TB$ intersect $AB$ and $AC$ at $P$ and $Q$ respectively.



      I need to prove that $TO$ is perpendicular to $PQ$.



      How do I prove it? I tried some simple angle chasing but couldn't come to any significant conclusion. The solution booklet however tells me to use the fact that the reflection of $O$ in $BC$, say $O'$ lies on the circumcircle of $triangle APQ$, how did he arrive at this conclusion?










      share|cite|improve this question











      $endgroup$




      I have a triangle $triangle ABC$, with circumcentre $O$, the reflection of $A$ through $BC$ gives me $T$. The perpendicular bisector of $TC$ and $TB$ intersect $AB$ and $AC$ at $P$ and $Q$ respectively.



      I need to prove that $TO$ is perpendicular to $PQ$.



      How do I prove it? I tried some simple angle chasing but couldn't come to any significant conclusion. The solution booklet however tells me to use the fact that the reflection of $O$ in $BC$, say $O'$ lies on the circumcircle of $triangle APQ$, how did he arrive at this conclusion?







      geometry






      share|cite|improve this question















      share|cite|improve this question













      share|cite|improve this question




      share|cite|improve this question








      edited Sep 18 '18 at 9:36









      limeeattack

      727




      727










      asked Sep 18 '18 at 6:33









      saisanjeevsaisanjeev

      1,025312




      1,025312






















          1 Answer
          1






          active

          oldest

          votes


















          1












          $begingroup$

          enter image description here



          Let $PR$ be the perpendicular bisector of side $TC$ , and $SQ$ be the perpendicular bisector of side $BT$ . Let them intersect at point $O’$.



          Observe that $O’$ is the circumcentre of $triangle BCT$ . It is thus the reflection of $O$ in $BC$.



          $$∠RPA + ∠SQA = ∠O’PA+O’QA = (360-90-2∠C-A)+(360-90-2∠B-A)
          $$
          $$=720-180-2(∠A+∠B+∠C) = 720 - 540 = 180 .$$



          Thus , the angles are supplementary. This implies $O’PAQ$ is a cyclic quadrilateral, and the points are concyclic.



          Join the diagonals and $OO’$ . Join $O’A$ and $OT$, to meet at $G$. Let $OO’$ meet $BC$ at $D$ . Let $AT$ meet $BC$ at $H$ .



          For convenience , Let $∠BAH = w , ∠HAG = y , ∠GAQ = x$ .



          Using the properties of cyclic quadrilaterals and by angle chasing , we get $∠BMP = y .$ Also observe that $ ∠HAG = ∠HTG = y$ and since $AT$ is parallel to $OO’ , ∠TOO’ = y.$



          Then , the line segments $PQ$ and $OT$ are inclined to $BC$ and $OO’$ respectively at an equal angle . This implies that the rotation of these line segments by that angle , will not change the angle between them , and the angle between them will equal $angle ODB$. But $∠ODB = 90.$



          Hence , OT is perpendicular to PQ






          share|cite|improve this answer











          $endgroup$













            Your Answer





            StackExchange.ifUsing("editor", function () {
            return StackExchange.using("mathjaxEditing", function () {
            StackExchange.MarkdownEditor.creationCallbacks.add(function (editor, postfix) {
            StackExchange.mathjaxEditing.prepareWmdForMathJax(editor, postfix, [["$", "$"], ["\\(","\\)"]]);
            });
            });
            }, "mathjax-editing");

            StackExchange.ready(function() {
            var channelOptions = {
            tags: "".split(" "),
            id: "69"
            };
            initTagRenderer("".split(" "), "".split(" "), channelOptions);

            StackExchange.using("externalEditor", function() {
            // Have to fire editor after snippets, if snippets enabled
            if (StackExchange.settings.snippets.snippetsEnabled) {
            StackExchange.using("snippets", function() {
            createEditor();
            });
            }
            else {
            createEditor();
            }
            });

            function createEditor() {
            StackExchange.prepareEditor({
            heartbeatType: 'answer',
            autoActivateHeartbeat: false,
            convertImagesToLinks: true,
            noModals: true,
            showLowRepImageUploadWarning: true,
            reputationToPostImages: 10,
            bindNavPrevention: true,
            postfix: "",
            imageUploader: {
            brandingHtml: "Powered by u003ca class="icon-imgur-white" href="https://imgur.com/"u003eu003c/au003e",
            contentPolicyHtml: "User contributions licensed under u003ca href="https://creativecommons.org/licenses/by-sa/3.0/"u003ecc by-sa 3.0 with attribution requiredu003c/au003e u003ca href="https://stackoverflow.com/legal/content-policy"u003e(content policy)u003c/au003e",
            allowUrls: true
            },
            noCode: true, onDemand: true,
            discardSelector: ".discard-answer"
            ,immediatelyShowMarkdownHelp:true
            });


            }
            });














            draft saved

            draft discarded


















            StackExchange.ready(
            function () {
            StackExchange.openid.initPostLogin('.new-post-login', 'https%3a%2f%2fmath.stackexchange.com%2fquestions%2f2921160%2fhow-to-prove-that-the-given-lines-are-perpendicular%23new-answer', 'question_page');
            }
            );

            Post as a guest















            Required, but never shown

























            1 Answer
            1






            active

            oldest

            votes








            1 Answer
            1






            active

            oldest

            votes









            active

            oldest

            votes






            active

            oldest

            votes









            1












            $begingroup$

            enter image description here



            Let $PR$ be the perpendicular bisector of side $TC$ , and $SQ$ be the perpendicular bisector of side $BT$ . Let them intersect at point $O’$.



            Observe that $O’$ is the circumcentre of $triangle BCT$ . It is thus the reflection of $O$ in $BC$.



            $$∠RPA + ∠SQA = ∠O’PA+O’QA = (360-90-2∠C-A)+(360-90-2∠B-A)
            $$
            $$=720-180-2(∠A+∠B+∠C) = 720 - 540 = 180 .$$



            Thus , the angles are supplementary. This implies $O’PAQ$ is a cyclic quadrilateral, and the points are concyclic.



            Join the diagonals and $OO’$ . Join $O’A$ and $OT$, to meet at $G$. Let $OO’$ meet $BC$ at $D$ . Let $AT$ meet $BC$ at $H$ .



            For convenience , Let $∠BAH = w , ∠HAG = y , ∠GAQ = x$ .



            Using the properties of cyclic quadrilaterals and by angle chasing , we get $∠BMP = y .$ Also observe that $ ∠HAG = ∠HTG = y$ and since $AT$ is parallel to $OO’ , ∠TOO’ = y.$



            Then , the line segments $PQ$ and $OT$ are inclined to $BC$ and $OO’$ respectively at an equal angle . This implies that the rotation of these line segments by that angle , will not change the angle between them , and the angle between them will equal $angle ODB$. But $∠ODB = 90.$



            Hence , OT is perpendicular to PQ






            share|cite|improve this answer











            $endgroup$


















              1












              $begingroup$

              enter image description here



              Let $PR$ be the perpendicular bisector of side $TC$ , and $SQ$ be the perpendicular bisector of side $BT$ . Let them intersect at point $O’$.



              Observe that $O’$ is the circumcentre of $triangle BCT$ . It is thus the reflection of $O$ in $BC$.



              $$∠RPA + ∠SQA = ∠O’PA+O’QA = (360-90-2∠C-A)+(360-90-2∠B-A)
              $$
              $$=720-180-2(∠A+∠B+∠C) = 720 - 540 = 180 .$$



              Thus , the angles are supplementary. This implies $O’PAQ$ is a cyclic quadrilateral, and the points are concyclic.



              Join the diagonals and $OO’$ . Join $O’A$ and $OT$, to meet at $G$. Let $OO’$ meet $BC$ at $D$ . Let $AT$ meet $BC$ at $H$ .



              For convenience , Let $∠BAH = w , ∠HAG = y , ∠GAQ = x$ .



              Using the properties of cyclic quadrilaterals and by angle chasing , we get $∠BMP = y .$ Also observe that $ ∠HAG = ∠HTG = y$ and since $AT$ is parallel to $OO’ , ∠TOO’ = y.$



              Then , the line segments $PQ$ and $OT$ are inclined to $BC$ and $OO’$ respectively at an equal angle . This implies that the rotation of these line segments by that angle , will not change the angle between them , and the angle between them will equal $angle ODB$. But $∠ODB = 90.$



              Hence , OT is perpendicular to PQ






              share|cite|improve this answer











              $endgroup$
















                1












                1








                1





                $begingroup$

                enter image description here



                Let $PR$ be the perpendicular bisector of side $TC$ , and $SQ$ be the perpendicular bisector of side $BT$ . Let them intersect at point $O’$.



                Observe that $O’$ is the circumcentre of $triangle BCT$ . It is thus the reflection of $O$ in $BC$.



                $$∠RPA + ∠SQA = ∠O’PA+O’QA = (360-90-2∠C-A)+(360-90-2∠B-A)
                $$
                $$=720-180-2(∠A+∠B+∠C) = 720 - 540 = 180 .$$



                Thus , the angles are supplementary. This implies $O’PAQ$ is a cyclic quadrilateral, and the points are concyclic.



                Join the diagonals and $OO’$ . Join $O’A$ and $OT$, to meet at $G$. Let $OO’$ meet $BC$ at $D$ . Let $AT$ meet $BC$ at $H$ .



                For convenience , Let $∠BAH = w , ∠HAG = y , ∠GAQ = x$ .



                Using the properties of cyclic quadrilaterals and by angle chasing , we get $∠BMP = y .$ Also observe that $ ∠HAG = ∠HTG = y$ and since $AT$ is parallel to $OO’ , ∠TOO’ = y.$



                Then , the line segments $PQ$ and $OT$ are inclined to $BC$ and $OO’$ respectively at an equal angle . This implies that the rotation of these line segments by that angle , will not change the angle between them , and the angle between them will equal $angle ODB$. But $∠ODB = 90.$



                Hence , OT is perpendicular to PQ






                share|cite|improve this answer











                $endgroup$



                enter image description here



                Let $PR$ be the perpendicular bisector of side $TC$ , and $SQ$ be the perpendicular bisector of side $BT$ . Let them intersect at point $O’$.



                Observe that $O’$ is the circumcentre of $triangle BCT$ . It is thus the reflection of $O$ in $BC$.



                $$∠RPA + ∠SQA = ∠O’PA+O’QA = (360-90-2∠C-A)+(360-90-2∠B-A)
                $$
                $$=720-180-2(∠A+∠B+∠C) = 720 - 540 = 180 .$$



                Thus , the angles are supplementary. This implies $O’PAQ$ is a cyclic quadrilateral, and the points are concyclic.



                Join the diagonals and $OO’$ . Join $O’A$ and $OT$, to meet at $G$. Let $OO’$ meet $BC$ at $D$ . Let $AT$ meet $BC$ at $H$ .



                For convenience , Let $∠BAH = w , ∠HAG = y , ∠GAQ = x$ .



                Using the properties of cyclic quadrilaterals and by angle chasing , we get $∠BMP = y .$ Also observe that $ ∠HAG = ∠HTG = y$ and since $AT$ is parallel to $OO’ , ∠TOO’ = y.$



                Then , the line segments $PQ$ and $OT$ are inclined to $BC$ and $OO’$ respectively at an equal angle . This implies that the rotation of these line segments by that angle , will not change the angle between them , and the angle between them will equal $angle ODB$. But $∠ODB = 90.$



                Hence , OT is perpendicular to PQ







                share|cite|improve this answer














                share|cite|improve this answer



                share|cite|improve this answer








                edited Dec 30 '18 at 9:13

























                answered Sep 29 '18 at 18:47









                SinπSinπ

                64511




                64511






























                    draft saved

                    draft discarded




















































                    Thanks for contributing an answer to Mathematics Stack Exchange!


                    • Please be sure to answer the question. Provide details and share your research!

                    But avoid



                    • Asking for help, clarification, or responding to other answers.

                    • Making statements based on opinion; back them up with references or personal experience.


                    Use MathJax to format equations. MathJax reference.


                    To learn more, see our tips on writing great answers.




                    draft saved


                    draft discarded














                    StackExchange.ready(
                    function () {
                    StackExchange.openid.initPostLogin('.new-post-login', 'https%3a%2f%2fmath.stackexchange.com%2fquestions%2f2921160%2fhow-to-prove-that-the-given-lines-are-perpendicular%23new-answer', 'question_page');
                    }
                    );

                    Post as a guest















                    Required, but never shown





















































                    Required, but never shown














                    Required, but never shown












                    Required, but never shown







                    Required, but never shown

































                    Required, but never shown














                    Required, but never shown












                    Required, but never shown







                    Required, but never shown







                    Popular posts from this blog

                    To store a contact into the json file from server.js file using a class in NodeJS

                    Redirect URL with Chrome Remote Debugging Android Devices

                    Dieringhausen